Question-and-Answer Resource for the Building Energy Modeling Community
Get started with the Help page
Ask Your Question
1

EnergyPlus FMU to Modelica

asked 2017-10-04 13:05:51 -0500

Charalampos Angelopoulos's avatar

updated 2018-05-21 15:11:03 -0500

I am using EnergyPlus to export and FMU and imported to Modelica. I am trying to replicate the model shown in http://simulationresearch.lbl.gov/fmu... which seems to work fairly ok. However, my next step is to include a PI-controller for the cooling mode. When i add the PI-controller the model doesn't seem to work properly as the results for the temperature are completely out of the setpoints (20C-heating and 27C-cooling). thank you in advance. I have attached also a screenshot of the PI-controller. I tried to tune the controller by adjusting the parameters K,Ti and although it has an effect on the results, still they are far away from the desired values. Could you please advice me on this issue? Thank you !!

image description

edit retag flag offensive close merge delete

2 Answers

Sort by ยป oldest newest most voted
2

answered 2017-10-04 17:11:07 -0500

I can't see anything in this annual plot other than that the room temperature switches multiple times between -100 and 220 degC. I suspect you have too high a control action (otherwise you would not get such extreme room temperatures) and your gains are such that the control is unstable. At this experimental stage, I see no reason for adding an integral term in the controller as integral terms can cause instability if incorrectly tuned, and all they do is avoid a steady state error. Your first task should be to get a stable P controller with reasonable actuation power, and then if you need to reduce the steady-state error, add integral action.

edit flag offensive delete link more
1

answered 2017-10-05 04:21:58 -0500

Charalampos Angelopoulos's avatar

This is the model i am working on! When i changed the parameters of the gain to 0.001 and Ti=300, then the model seems to work better. I also used the P-Controller as you suggested, and the results are similar to the PI case. For the P-Controller k=0.01. My question is if those numbers seem reasonable or are too small.

image description

edit flag offensive delete link more

Your Answer

Please start posting anonymously - your entry will be published after you log in or create a new account.

Add Answer

Careers

Question Tools

1 follower

Stats

Asked: 2017-10-04 13:05:51 -0500

Seen: 2,603 times

Last updated: Oct 05 '17